A researcher at the University of Washington medical school believes that energy drink consumption may increase heart rate. Suppose it is known that heart rate (in beats per minute) is normally distributed with an average of 70 bpm for adults. A random sample of 25 adults was selected and it was found that their average heartbeat was 73 bpm after energy drink consumption, with a standard deviation of 7 bpm. In order to test belief at the 10% significance level, determine P-value for the test.

Answers

Answer 1

Answer:

Step-by-step explanation:

Given that:

mean μ = 70

sample size = 25

sample mean = 73

standard deviation = 7

level of significance = 0.10

The null hypothesis and the alternative hypothesis can be computed as follows:

[tex]\mathtt{H_o : \mu = 70} \\ \\ \mathtt{H_1 : \mu > 70 }[/tex]

The z score for this statistics can be calculated by using the formula:

[tex]z = \dfrac{X- \mu}{\dfrac{\sigma}{\sqrt{n}}}[/tex]

[tex]z = \dfrac{73- 70}{\dfrac{7}{\sqrt{25}}}[/tex]

[tex]z = \dfrac{3}{\dfrac{7}{5}}[/tex]

[tex]z = \dfrac{3 \times 5}{{7}{}}[/tex]

z = 2.143

At level of significance of 0.10

degree of freedom = n -1

degree of freedom = 25 - 1

degree of freedom = 24

The p - value from the z score at   level of significance of 0.10 and degree of freedom of 24 is:

P - value = 1 - (Z < 2.143)

P - value =  1 - 0.9839

P - value =  0.0161

Decision Rule: since P value is lesser than the level of significance, we reject the null hypothesis.

Conclusion: We conclude that energy drink consumption increases heart rate.


Related Questions

Quick! Andrew has to play 15 games in a chess tournament. At some point during the tournament he has won half of the games he has played, he has lost one-third of the games he has played and two have ended in a draw. How many games has Andrew still to play?

Answers

[tex]x[/tex] - the number of the games he played

[tex]\dfrac{x}{2}[/tex] - the number of the games he won

[tex]\dfrac{x}{3}[/tex] - the number of the games he lost

[tex]x=\dfrac{x}{2}+\dfrac{x}{3}+2\Big|\cdot6\\6x=3x+2x+12\\x=12[/tex]

[tex]15-12=3[/tex]

so, he has still 3 games to play

Tessa’s employee benefits include family health care coverage. She contributes 18% of the cost. Tessa gets paid biweekly and $108.00 is taken out of each paycheck for family health care coverage. How much does her employer contribute annually for the family coverage? Clearly show your work.

Answers

The answer is $12792

Explanation:

It is known Tessa pays $108.00 to contribute to family coverage every two weeks and this represents 18% of the total payment. This implies the employer pays the 82% missing (100% - 18% = 82%). Additionally, with this information, it is possible to know the amount the employer has to pay every two weeks that represents 82%. The process is shown below:

1. Write the values you know and use x to represent the value you need to find

108 = 18        

  x =   82      

3. Cross multiply

x 18 = 8856

4. Find  the value of x by solving this simple equation

x = 8856 ÷ 18

x = 492 - Amount the employer pays every two weeks for Tessa's family coverage

Now that we know the money the employer pays every two weeks, it is possible to calculate the annual amount of money. Follow the process below.

1. Consider one year has a total of 52 weeks and divide this number of weeks by 2 because the payment for the family coverage occurs every 2 weeks

52 ÷ 2 = 26

2. Finally, multiply the money paid by the employer every two weeks by 26

26 weeks x $492 = $12792- This is the total the employer pays annually

Find the area of the shape shown below.
3.5
2
2

Answers

Answer:

26.75 units²

Step-by-step explanation:

Cube Area: A = l²

Triangle Area: A = 1/2bh

Step 1: Find area of biggest triangle

A = 1/2(3.5)(2 + 2 + 5)

A = 1.75(9)

A = 15.75

Step 2: Find area of 2nd biggest triangle

A = 1/2(5)(2)

A = 1/2(10)

A = 5

Step 3: Find area of smallest triangle

A = 1/2(2)(2)

A = 1/2(4)

A = 2

Step 4: Find area of cube

A = 2²

A = 4

Step 5: Add all the values together

A = 15.75 + 5 + 2 + 4

A = 20.75 + 2 + 4

A = 22.75 + 4

A = 26.75

Use the Pythagorean theorem to find the length of the hypotenuse in the triangle shown below.4,3

Answers

Answer:

5

Step-by-step explanation:

a^2 + b^2 = c^2

4^2 + 3^2 = c^2

16 + 9 = c^2

25 = c^2

c = 5

Answer:

5

Step-by-step explanation:

[tex]Hypotenuse = ?\\Opposite = 4\\Adjacent = 3\\\\Pythagoras \: Theorem ;\\\\Hypotenuse^2 =Opposite^2+Adjacent ^2\\\\Hypotenuse^2 = 4^2 +3^2\\\\Hypotenuse^2 = 16+9\\\\Hypotenuse^2 = 25\\\\\sqrt{Hypotenuse^2}=\sqrt{25} \\Hypotenuse = 5[/tex]

Find the volume of the cylinder. Round your answer to the nearest tenth.

Answers

Answer:

716.75 m^3

Step-by-step explanation:

Volume of a cylinder:

=> PI x R^2 x H

H = Height

R = Radius

=> PI x 3.9^2 x 15

=> PI x 15.21 x 15

=> PI x 228.15

=> 228.15 PI

           or

=> 228.15 x 3.14159

=> 716.75 m^3

the length of a mathematical text book the is approximately 18.34cm and its width is 11.75 calculate ?​

the approximate perimeter of the front cover?

the approximate area of the front cover of the book?

Answers

Answer:

Perimeter=60.18cm

Area=215.495cm^2

Step-by-step explanation:

Given:

Length of book=18.34cm

Breadth=11.75cm

Solution:

Perimeter=2(l +b)

P=2(18.34+11.75)

P=2 x 30.09

P=60.18cm

Area=l x b

A=18.34 x 11.75

A=215.495 cm^2

Thank you!

Jesse bought 3 T-shirts for $6 each and 4 T-shirts for $5 each. What expression can you use to describe what Jesse bought?

Answers

(3x6) + (4x5). Is your answer for this question

the product of two consecutive positive integer is 306​

Answers

Answer:

[tex]\Large \boxed{\sf 17 \ and \ 18}[/tex]

Step-by-step explanation:

The product means multiplication.

There are two positive consecutive integers.

Let the first positive consecutive integer be x.

Let the second positive consecutive integer be x+1.

[tex](x) \times (x+1) =306[/tex]

Solve for x.

Expand brackets.

[tex]x^2 +x =306[/tex]

Subtract 306 from both sides.

[tex]x^2 +x -306=306-306[/tex]

[tex]x^2 +x -306=0[/tex]

Factor left side of the equation.

[tex](x-17)(x+18)=0[/tex]

Set factors equal to 0.

[tex]x-17=0[/tex]

[tex]x=17[/tex]

[tex]x+18=0[/tex]

[tex]x=-18[/tex]

The value of x cannot be negative.

Substitute x=17 for the second consecutive positive integer.

[tex](17)+1[/tex]

[tex]18[/tex]

The two integers are 17 and 18.

The product of two consecutive positive integers is 306.

We need to find the integers

solution : Let two consecutive numbers are x and (x + 1)

A/C to question,

product of x and (x + 1) = 306

⇒x(x + 1) = 306

⇒x² + x - 306 = 0

⇒ x² + 18x - 17x - 306 = 0

⇒x(x + 18) - 17(x + 18) = 0

⇒(x + 18)(x - 17) = 0⇒ x = 17 and -18

so x = 17 and (x +1) = 18

Therefore the numbers are 17 and 18.

Hope it helped u if yes mark me BRAINLIEST

TYSM!

A speedboat moves at a rate of 25 km/hr in still water. How long will it take
someone to ride the boat 87 km downstream if the river's current moves at a rate of
4 km/hr?

Answers

Answer:

3 hours

Step-by-step explanation:

Downstream, the speeds add up:

25 + 4 = 29 km/h

It will take:

87/29= 3 hrs

To ride 87 km.

What is the solution to X+9 = 24?
A. x = 33
B. x= 15
C. x= 18
D. x= 9​

Answers

Answer:

X+9=24

Or,x=24-9

:.x=15

Step-by-step explanation:

Answer:

B. x=15

Step-by-step explanation:

To find the solution to the equation, we must get x by itself on one side of the equation.

[tex]x+9=24[/tex]

9 is being added to x. The inverse of addition is subtraction. Subtract 9 from both sides of the equation.

[tex]x+9-9=24-9[/tex]

[tex]x=24-9[/tex]

[tex]x=15[/tex]

Let's check our solution. Plug 15 in for x.

[tex]x+9=24 (x=15)[/tex]

[tex]15+9=24[/tex]

[tex]24=24[/tex]

This checks out, so we know our solution is correct. The answer is B. x=15

point estimate A sample of 81 observations is taken from a normal population with a standard deviation of 5. The sample mean is 40. Determine the 95% confidence interval for the population mean

Answers

Answer:

The 95 percent Confidence Interval is for the population is (38.911 , 41.089)

Step-by-step explanation:

To solve the above question, we would be making use of the confidence interval formula:

Confidence Interval = Mean ± z score × σ/√n

In the above question,

Mean = 40

σ = Standard deviation = 5

n = number of samples = 81

Confidence Interval = 95%

The z score for a 95% confidence interval = 1.96

Therefore, the confidence interval =

= 40 ± 1.96 (5/√81)

= 40 ± 1.96(5/9)

= 40 ± 1.0888888889

Confidence Interval

a)40 + 1.0888888889

= 41.0888888889

Approximately = 41.089

b ) 40 - 1.0888888889

= 38.911111111

Approximately = 38.911

Therefore, the 95 percent Confidence Interval is for the population is (38.911 , 41.089)

If Company X has 1600 employees and 80% of those employees have attended the warehouse training course how many employees have yet to attend?

Answers

Answer:

320

Step-by-step explanation:

Total no of employees = 1600

% of employees attended the training = 80%

no. of employee who attended the training = 80/100* 1600 = 1280

No. of employees who are yet to attend the training = Total no of employees - no. of employee who attended the training =  1600-1280 = 320

Thus, 320 employees have yet to attend the training

Which quadratic equation would be used to solve for the unknown dimensions?

0 = 2w2
512 = w2
512 = 2w2
512 = 2l + 2w

Answers

Answer:

C

Step-by-step explanation:

Answer:

C: 512 = 2w2

Step-by-step explanation:

on edge

Evaluate
1+5.3
2
please answer quickly ​

Answers

Answer:

1+5.3=6.3

Step-by-step explanation:

not sure what your asking for with the 2

explain what your looking for with the 2 and maybe we can help you further

(I have to do it the way I did it because the 2 in the question is confusing)

Answer:

For expression 1 + 5.32: 6.32

For expression 1 + 5.3 × 2: 11.6

Step-by-step explanation:

If the expression is 1 + 5.32:

Add 1 to 5.32: 1 + 5.32 = 6.32

If the expression is 1 + 5.3 × 2:

5.3 × 2 = 10.6Plug in 10.6: 1 + 10.61 + 10.6 = 11.6

60 is x% of 12. Find the value of x.

Answers

Answer:

20

Step-by-step explanation:

We can set up a percentage proportion to find the value of x.

[tex]\frac{12}{x} = \frac{60}{100}[/tex]

Now we cross multiply:

[tex]100\cdot12=1200\\\\1200\div60=20[/tex]

Hope this helped!

Select the correct answer from each drop-down menu.
The function f is given by the table of values as shown below.

x 1 2 3 4 5
f(x) 13 19 37 91 253
Use the given table to complete the statements.

The parent function of the function represented in the table is
.

If function f was translated down 4 units, the
-values would be
.

A point in the table for the transformed function would be
.

Answers

Answer:

3^x9, 15, 33, 87, 249(4, 87) for example

Step-by-step explanation:

a) First differences of the f(x) values in the table are ...

  19 -13 = 6, 37 -19 = 18, 91 -37 = 54, 253 -91 = 162

The second differences are not constant:

  18 -6 = 12, 54 -18 = 36, 162 -54 = 108

But, we notice that both the first and second differences have a common ratio. This is characteristic of an exponential function. The common ratio is 18/6 = 3, so the parent function is 3^x.

__

b) Translating a function down 4 units subtracts 4 from each y-value. The values of f(x) in the table would be ...

  9, 15, 33, 87, 249

__

c) The x-values of the function stay the same for a vertical translation, so the points in the table of the transformed function are ...

  (x, f(x)) = (1, 9), (2, 15), (3, 33), (4, 87), (5, 249)

Answer: I think this is it:

The parent function of the function represented in the table is exponential. If function f was translated down 4 units, the f(x)-values would be decreased by 4. A point in the table for the transformed function would be (4,87)

Step-by-step explanation: I got it right on Edmentum!

How to graph the line y=4/3x

Answers

Answer:

make a table of values

Step-by-step explanation:

then plot using those values

The required graph has been attached which represents the line y = 4/3x

What is a graph?

A graph can be defined as a pictorial representation or a diagram that represents data or values.

We have been given the equation of a line below as:

y = 4/3x

Rewrite in slope-intercept form.

y = (4/3)x

Use the slope-intercept form to discover the slope and y-intercept.

Here the slope is 4/3 and  y-intercept = (0, 0)

Any line can be graphed using two points. Select two x values, and plug them into the equation to find the corresponding y values.

When substitute the value of x = 0, then the value of y = 0, and When substitute the value of x = 3, then the value of y = -4,

Hence, the graph represents the line y = 4/3x

Therefore, the required graph of the line y=4/3x will be shown in the as attached file.

Learn more about the graphs here:

brainly.com/question/16608196

#SPJ2

Please help with this

Answers

Answer:

A

Step-by-step explanation:

● first one:

The diagonals of a rhombus are perpendicular to each others wich means that they form four right angles.

STP is one of them so this statement is true.

● second one:

If ST and PT were equal this would be a square not a rhombus.

● third one:

If SPQ was a right angle, this woukd be a square.

● fourth one:

Again if the diagonals SQ and PR were equal, this would be a square.

Put the following equation of a line into slope-intercept form, simplifying all
fractions.
3x + 3y = -9

Answers

Answer:

[tex]y = -x - 3[/tex]

Step-by-step explanation:

We are trying to get the equation [tex]3x + 3y = -9[/tex] into the form [tex]y = mx+b[/tex], aka slope-intercept form.

To do this we are trying to isolate y.

[tex]3x + 3y = -9[/tex]

Subtract 3x from both sides:

[tex]3y = -9 - 3x[/tex]

Rearrange the terms:

[tex]3y = -3x - 9[/tex]

Divide both sides by 3:

[tex]y = -x - 3[/tex]

Hope this helped!

Question on Statistics and Confidence Intervals
A field test for a new exam was given to randomly selected seniors. The exams were graded, and the sample mean and sample standard deviation were calculated. Based on the results, the exam creator claims that on the same exam, nine times out of ten, seniors will have an average score within 5% of 75%.
Is the confidence interval at 90%, 95%, or 99%? What is the margin of error? Calculate the confidence interval and explain what it means in terms of the situation. (10 points)

Answers

The phrasing "nine times out of ten" means 9/10 = 0.90 = 90% is the confidence level. We're confident 90% of the time that the confidence interval captures the population parameter we're after (in this case mu = population mean)

The portion "have an average score within 5% of 75%" means that 75% = 0.75 is the center of the confidence interval, and it goes as low as 0.75 - 0.05 = 0.70 and as high as 0.75 + 0.05 = 0.80

This confidence interval is from 70% to 80%, meaning that nine times out of ten, we're confident that the average score is between 70% and 80%

We write the confidence interval as (0.70, 0.80). It's common to use the notation (L, U) to indicate the lower (L) and upper (U) boundaries. You might see the notation in the form L < mu < U. If so, then it would be 0.70 < mu < 0.80; either way they mean the same thing.

The margin of error is 0.05 as its the 5% radius of the interval. It tells us how far the most distant score is from the center (75%)

=========================================

In summary, we have these answers

confidence level = 90%margin of error = 5% = 0.05confidence interval = (0.70, 0.80)interpretation = We're 90% confident that the average exam score is between 0.70 and 0.80

The base of a triangle is 4 cm greater than the
height. The area is 30 cm. Find the height and
the length of the base
h
The height of the triangle is
The base of the triangle is​

Answers

Answer:

Step-by-step explanation:

Formula for area of a triangle:

Height x Base /2

Base (b) = h +4

Height = h

h + 4 x h /2 = 30cm

=> h +4 x h = 60

=> h+4h =60

=> 5h = 60

=> h = 12

Height = 12

Base = 12 +4 = 16

The function fix) = (x - 4)(x - 2) is shown.
What is the range of the function?
8
all real numbers less than or equal to 3
all real numbers less than or equal to -1
all real numbers greater than or equal to 3
all real numbers greater than or equal to - 1
6
2
16
2
14
COL
40
8
G D​

Answers

Answer:

The range of the function f(x)= (x-4)(x-2) is all real numbers greater than or equal to -1

Step-by-step explanation:

Question 36 of 40
The distance of a line bound by two points is defined as
L?
O A. a line segment
B. a ray
O
c. a plane
O D. a vertex
SUBMI​

Answers

Answer:

A. a line segment

Step-by-step explanation:

a ray is directing in one dxn, and has no end pointa plane is a closed, so more than 2 points a vertex is a single point itself

#2. Given the following conditional statement; which answer is
represents the biconditional statement: "If Mr. Anderson is a ninja, then
he can run like Naruto."
Mr. Anderson is a ninja iff he can run like Naruto.
Mr. Anderson can run like Naruto iff he is a ninja.
Mr. Anderson is Naruto iff he can run like a ninja.

Answers

Answer:

Mr. Anderson can run like Naruto iff he is a ninja.

Step-by-step explanation:

This is because, in the statement "If Mr. Anderson is a ninja, then  he can run like Naruto.", the sub-statement, "he can run like Naruto.", depends on the sub-statement 'If Mr Anderson is a Ninja'. This means that although Mr. Anderson is a Ninja, he can only run like Naruto if and only if he is a Ninja implying that if Mr Anderson is not a Ninja, he cannot run like Naruto.

So, Mr Anderson can run like Naruto iff he is a Ninja is the correct answer

Answer:

1

Step-by-step explanation:

Try to get to every number from 1 to 10 using four 4's and any number of arithmetic operations (+, −, ×, ÷). You may also you parentheses.

Answers

Answer:

Step-by-step explanation:

1. 4/4+4-4=1

2. 4/4+4/4=2

3. 4+4/4-4=3

4. 4 × (4 − 4) + 4=4

5. (4 × 4 + 4) / 4=5

6. 44 / 4 − 4=6

7. 4+4-4/4=7

8. 4+4+4-4=8

9. 4+4+4/9=9

10. 44 / 4.4=10

Answer:

1 = (4 x 4)/(4 x 4) or  (4 + 4)/(4 + 4) or  (4 / 4) x (4 / 4) or  (4 / 4)/(4 / 4)  

2= (4 x 4)/(4 + 4) or 4 / ((4+4)/4)

3= (4 + 4 + 4)/4 or (4 x 4 - 4)/4

4 = 4 - (4 - 4)/4

5 = (4 x 4 + 4)/4

6 = 4 + (4 + 4)/4

7 = 4 - (4/4) + 4

8 = 4 + (4 x 4)/4

9 = 4 + 4 + (4/4)

10 - I tried the best. You might need ! or sqrt operator to get 4.

Updated:

I forgot we could use 4, 44, 444, or 4444, so that 10 could be expressed as:

10 = (44 - 4)/4

If “n” is a positive integer divisible by 3 and n is less than or equal to 44, then what is the highest possible value of n?

Answers

Answer:

Step-by-step explanation:

positive integer divisible by 3 includes

3,6,9,12,15,18,21,24,27,30,33,36,39,42,45...

less than highest possible value is 42

Identify whether the sampling method is simple random, systematic, stratified, cluster, or convenience. Explain.

In a nationwide study of registered voters conducted by The New York Times, 390 people are randomly selected out of those registered as Republicans, 430 people are randomly selected out of those registered as Democrats, and 180 people are randomly selected out of those registered as Independents.

Answers

Answer: stratified

Step-by-step explanation:

In stratified sampling, you divide the population into subgroups, or strata, with similar characteristics, like here we have divided the population into subgroups that depend on their political alignment. This is used when you can expect that the results have a noticeable variation between the different subgroups. Usually, you want to have the same number of population for eac subgroup, but sometimes it is hard for different reasons (not enough people in one subgroup, for example)

In cluster sampling we also use subgroups, but the subgroup itself is the unit of the sampling, while in this case, we are randomly selecting individuals of the given subgroups.

So this would be a "stratified sampling".

Which expression is equal to 7 times the sum of a number and 4

Answers

Answer:

7(n + 4)

Step-by-step explanation:

Represent the number by n.  Then the verbal expression becomes

7(n + 4).

Can somebody explain how trigonometric form polar equations are divided/multiplied?

Answers

Answer:

Attachment 1 : Option C

Attachment 2 : Option A

Step-by-step explanation:

( 1 ) Expressing the product of z1 and z2 would be as follows,

[tex]14\left[\cos \left(\frac{\pi \:}{5}\right)+i\sin \left(\frac{\pi \:\:}{5}\right)\right]\cdot \:2\sqrt{2}\left[\cos \left(\frac{3\pi \:}{2}\right)+i\sin \left(\frac{3\pi \:\:}{2}\right)\right][/tex]

Now to solve such problems, you will need to know what cos(π / 5) is, sin(π / 5) etc. If you don't know their exact value, I would recommend you use a calculator,

cos(π / 5) = [tex]\frac{\sqrt{5}+1}{4}[/tex],

sin(π / 5) = [tex]\frac{\sqrt{2}\sqrt{5-\sqrt{5}}}{4}[/tex]

cos(3π / 2) = 0,

sin(3π / 2) = - 1

Let's substitute those values in our expression,

[tex]14\left[\frac{\sqrt{5}+1}{4}+i\frac{\sqrt{2}\sqrt{5-\sqrt{5}}}{4}\right]\cdot \:2\sqrt{2}\left[0-i\right][/tex]

And now simplify the expression,

[tex]14\sqrt{5-\sqrt{5}}+i\left(-7\sqrt{10}-7\sqrt{2}\right)[/tex]

The exact value of [tex]14\sqrt{5-\sqrt{5}}[/tex] = [tex]23.27510\dots[/tex] and [tex](-7\sqrt{10}-7\sqrt{2}\right))[/tex] = [tex]-32.03543\dots[/tex] Therefore we have the expression [tex]23.27510 - 32.03543i[/tex], which is close to option c. As you can see they approximated the solution.

( 2 ) Here we will apply the following trivial identities,

cos(π / 3) = [tex]\frac{1}{2}[/tex],

sin(π / 3) = [tex]\frac{\sqrt{3}}{2}[/tex],

cos(- π / 6) = [tex]\frac{\sqrt{3}}{2}[/tex],

sin(- π / 6) = [tex]-\frac{1}{2}[/tex]

Substitute into the following expression, representing the quotient of the given values of z1 and z2,

[tex]15\left[cos\left(\frac{\pi \:}{3}\right)+isin\left(\frac{\pi \:\:}{3}\right)\right] \div \:3\sqrt{2}\left[cos\left(\frac{-\pi \:}{6}\right)+isin\left(\frac{-\pi \:\:}{6}\right)\right][/tex] ⇒

[tex]15\left[\frac{1}{2}+\frac{\sqrt{3}}{2}\right]\div \:3\sqrt{2}\left[\frac{\sqrt{3}}{2}+-\frac{1}{2}\right][/tex]

The simplified expression will be the following,

[tex]i\frac{5\sqrt{2}}{2}[/tex] or in other words [tex]\frac{5\sqrt{2}}{2}i[/tex] or [tex]\frac{5i\sqrt{2}}{2}[/tex]

The solution will be option a, as you can see.

On a coordinate plane, a line goes through (negative 3, 3) and (negative 2, 1). A point is at (4, 1). What is the equation, in point-slope form, of the line that is parallel to the given line and passes through the point (4, 1)? y − 1 = −2(x − 4) y – 1 = Negative one-half(x – 4) y – 1 = One-half(x – 4) y − 1 = 2(x − 4)

Answers

Answer:

y - 1 = -2(x - 4).

Step-by-step explanation:

First, we need to find the slope. Two sets of coordinates are (-3, 3), and (-2, 1).

(3 - 1) / (-3 - -2) = 2 / (-3 + 2) = 2 / (-1) = -2.

The line will be parallel to the given line, so the slope is the same.

Now that we have a point and the slope, we can construct an equation in point-slope form.

y1 = 1, x1 = 4, and m = -2.

y - 1 = -2(x - 4).

Hope this helps!

The slope of the line passing  parallel to the given line and passes through the point (4, 1) is y = -2x + 9

The equation of a straight line is given by:

y = mx + b

where y, x are variables, m is the slope of the line and b is the y intercept.

The slope of the line passing through the points (-3,3) and  (-2,1) is:

[tex]m=\frac{y_2-y_1}{x_2-x_1} \\\\m=\frac{1-3}{-2-(-3)} \\\\m=-2[/tex]

Since both lines are parallel, hence they  have the same slope (-2). The line passes through (4,1). The equation is:

[tex]y-y_1=m(x-x_1)\\\\y-1=-2(x-4)\\\\y=-2x+9[/tex]

Find out more at: https://brainly.com/question/18880408

Other Questions
for 0 Given the following hypotheses: H0: = 490 H1: 490 A random sample of 15 observations is selected from a normal population. The sample mean was 495 and the sample standard deviation 9. Using the 0.01 significance level: a.) State the decision rule. b.) Compute the value of the test statistic. c.) What is your decision regarding the null hypothesis? Antiono le a su madre ahroa estate castigado por un mes Escribe las manifestaciones culturales de tu localidad y establece las semajanzas y diferencias con las estudiadas en el caso del continente asitico.ayuda. anyone know how to solve a functions equation such as x^2-x-x Determine if the process appears to be within statistical control. If not, state the reason why not. a. It does not appear to be within statistical control because there is an upward shift. b. It appears to be within statistical control. c. It does not appear to be within statistical control because there is an upward trend. d. It does not appear to be within statistical control because there is increasing variation. Section AMultiple ChoiceRead the questions carefully and draw a circle around the letter of the correct answer.1. Where were these ceramic figures of a Bactrian camel and groom once placed?A. in the palaces of Chinese emperorsB. in the huts of poor villagersC. in the tombs of wealthy aristocrats or merchantsD. in the homes of wealthy aristocrats or merchants For what real numbers x is x 2 10 x + 25 negative? 1. At the end of one school day a teacher had 17 crayons left. The teacher rememberedgiving out 14 crayons in the morning, getting 12 crayons back at recess, and giving out11 crayons after lunch. How many crayons did the teacher have at the start of theday? List the six major areas of emphasis, other than melancholy, that are characteristic of the romantic philosophy. You short-sell 100 shares of Tuckerton Trading Co., now selling for $44 per share. What is your maximum possible gain, ignoring transactions cost You and your best friend are both on the swim team. You want to beat your friend at the next swim meet so you decide to swim 151515 minutes longer than she does one day at practice. Write an equation for the number of minutes you swim, yyy, when your friend swims xxx number of minutes. what is a hypothesis reffered to as after being verified by a large number or independent experiments A radar installation operates at 9000 MHz with an antenna (dish) that is 15 meters across. Determine the maximum distance (in kilometers) for which this system can distinguish two aircraft 100 meters apart. Describe various ways that knowledge management systems could help firms with sales and marketing or with manufacturing and production. 5/7 minus 2/9 please The Treasury bill rate is 4% and the market risk premium is 7%. Project Beta Internal rate of return %P 1.0 14Q 0 6R 2.0 18S 0.4 7T 1.6 20Required:a. What are the project costs of capital for new ventures with betas of 0.75 and 1.75?b. Which of the following capital investments have positive NPVs?1. P2. Q3. R4. S5. T 6=m/8 whats does m equal? Please give me the correct answer Answer ASAP, Will give brainliest!!